LSAT and Law School Admissions Forum

Get expert LSAT preparation and law school admissions advice from PowerScore Test Preparation.

 Administrator
PowerScore Staff
  • PowerScore Staff
  • Posts: 8917
  • Joined: Feb 02, 2011
|
#22814
Complete Question Explanation

Weaken. The correct answer choice is (B)

The dialogue between Randy and Marion presents two competing explanations for the mayor's decision to get Azco time on a community channel. According to Randy, her actions demonstrate a concern for keeping a diversity of news programming in the city; Marion, on the other hand, suspects the mayor's motives were to reward Azco's owner, a political supporter of hers.

Essentially, we are dealing with two competing causes for the same effect:
  • ..... ..... Cause ..... ..... ..... ..... ..... Effect

    Randy: ..... concern for diversity

    ..... ..... ..... ..... ..... ..... .......... Azco time on community channel

    Marion: ..... cruder motives
Because the question is asking us to take Randy's side, our goal will be twofold: to strengthen Randy's causal relationship and/or weaken Marion's. In other words, we need to find an answer choice that reaffirms the view that the mayor's actions were benevolent and not driven by sinister motives.

Answer choice (A): At first glance, this seems like an attractive answer, as it implies that the Azco owner expected no political reward for his support. However, whether or not the reward was expected at the receiving end is irrelevant to the mayor's decision to extend it. Maybe the mayor mistakenly thought Azco expected a pay-back? Or maybe she rewarded him despite the lack of any expectations of receiving a reward? Either way, this answer choice does not weaken Marion's argument and is therefore incorrect.

Answer choice (B): This is the correct answer choice. If the mayor used her influence to get time on a community channel for another news service, whose owner supported the mayor's opponent in the last election, this would strongly suggest that her support in the present case is not fully explainable by cruder motives. In other words, this answer choice presents a counterexample to Marion's cause-and-effect hypothesis: it shows a scenario where the effect occurs (the mayor using her influence to get time on the channel for another news service), but the cause does not (no reason to reward a political opponent). Because this answer choice weakens Marion's argument, it is the correct answer.

Answer choice (C): Azco's news coverage of the mayor is irrelevant to Marion's argument. Furthermore, if it's never been judged to be biased, then perhaps it is biased — thus strengthening Marion's argument, not weakening it.

Answer choice (D): This answer choice implies that the mayor's decision can be explained by political motives, albeit not exactly the same as those suspected by Marion. Because it strengthens the "cruder motives" explanation for the mayor's action, this answer choice does the exact opposite of what is needed and is incorrect.

Answer choice (E): The number of viewers who watch Mega Cable TV is irrelevant to either argument. This answer choice is incorrect.
 tejinder12
  • Posts: 8
  • Joined: Jul 16, 2017
|
#37514
I got the right answer, but when i first read B i was thinking of the fact that the major helped the other news service get on the channel so that it would influence their vote and make them vote him/her
 Francis O'Rourke
PowerScore Staff
  • PowerScore Staff
  • Posts: 471
  • Joined: Mar 10, 2017
|
#37687
Hi Tejinder,

There is a some ambiguity in the tense in answer choice (B). I originally read this answer as stating that after the election, the mayor used her influence to help a channel that was previously opposed to her during the election.

Your reading of answer choice (B) tells us that the mayor used her influence at some point before the election to help out the other news service in hopes that that news service would become more supportive of her electoral bid. This interpretation is possible, but it would also provide an objection to Marion's argument.

Marion states that the mayor was motivated to reward her political supporters. If we point to her actions according to your interpretation of answer choice (B), we would still counter Marion's conclusion. It would thus be possible that her actions in regards to Azco were not to reward a friend, but a savvy political move. This alternative interpretation of answer choice (B) does not tell us that the mayor is altruistic, but it does provide some evidence that her actions were something other than a reward, as Marion claims.
 martinbeslu
  • Posts: 49
  • Joined: Aug 09, 2017
|
#42451
I am in agreement with the statement in the initial explanation but I see other answer choices being stronger than answer choice B:

“Because the question is asking us to take Randy's side, our goal will be twofold: to strengthen Randy's causal relationship and/or weaken Marion's.”

Answer choice A tells us that the owner of Azco supported the mayor because he liked her political agenda, not for a reward. We have to take the statements given in the answer choices as true according to the question stem. It doesn’t say that Azco didn’t think the mayor expected a reward. It explicitly states that there was no expected reward. If no reward was expected that has to make it less likely that it was her reason for doing it. Sure, it’s possible that the mayor could reward her anyway but Marion’s argument has been weakened significantly. This directly counter’s Marion’s argument that the mayor was rewarding Azco’s owner. Our job is to weaken Marion’s argument, not to prove it false, and that is exactly what this answer choice does.

Answer choice B tells us the mayor also used her influence “to get time on a community channel for another news service…” However, over and over we have learned that the test makers use examples that are outside the scope to trick us into picking bad answer choices. There are countless examples of this trick in the questions we’ve seen so far. No matter what happened in the situation with “another news service”, this answer choice is clearly outside the scope of the mayors dealings with Azco. This is a logical flaw.
The argument is about the mayor rewarding Azco’s owner, not about what she did for another news service. This answer choice is irrelevant at best or it possibly even strengthen’s Marion’s argument that the mayor sometimes makes these decisions for political (crude) motives.

For example, if you were making the argument that I went to Taco Bell today I couldn’t weaken that argument simply by saying that I went to Subway last week. That would be entirely outside the scope of an argument about where I went today. We could also interpret the statement in support of Marion just as easily as we could interpret it in support of Randy and therefore this seems like a very poor answer choice.

Answer choice D looks like the strongest answer. As stated at the beginning, our only job is to strengthen Randy's causal relationship and/or weaken Marion’s. Marion’s position is that the mayor was rewarding Azco’s owner and therefore had crude motives. However, answer choice D tells us that many people’s jobs depend on Azco and this is a possible reason the mayor used her influence. This seems like a very good reason for the mayor to support Azco. Isn’t this what a mayor is supposed to do? I don’t see how this could be twisted so far as to say that saving people’s jobs is crude because then they are more likely to support her. This isn’t a “crude” motive. This answer choice proves that there was an alternate cause and therefore weakens Marion’s argument.
 Jennifer Janowsky
PowerScore Staff
  • PowerScore Staff
  • Posts: 90
  • Joined: Aug 20, 2017
|
#42472
Hi, martinbeslu!

In this case, it is the Mayor's motives that are being argued about, not the company's. Just because the company did not expect a reward, as suggested in (A), does not say anything about the motives of the Mayor.

However, the motives of the Mayor are called into question by (B): In this case, she is still supporting a small news station, but now the station has a history of actively supporting her competitor. This shows her completing the same action without benefit (and even with cost), therefore showing that the motives behind that action are to keep news diverse. It shows a situation in which the effect is isolated from the alleged cause.

It is important to remember that out of scope answers are not to be eliminated outright for Strengthen/Weaken questions. In Strengthen/Weaken questions, additional information may be appropriate so long as it harms or hurts the conclusion.

For (D), we must remember that the goal here is not merely to hurt Marion's argument, but also to help Randy's. Marion's argument is undoubtedly hurt, as a new motive is suggested: helping the Mayor's constituents. However, this also shows that the Mayor was not merely "demonstrating her concern for keeping a diversity of news programming in the city" as Randy had suggested, hurting his argument as well.

I hope this answers your questions!
 lsat.bea
  • Posts: 4
  • Joined: May 24, 2020
|
#76599
I chose (e) for this one. My thinking was that if the number of people watching Mega Cable's programming has decreased, then perhaps the mayor thought that that might be ameliorated by ensuring that a diverse array of news programming is available to peak interest in potential viewers. Hence, the mayor was actually concerned about keeping a diversity of news programming in the city, as her decision to get Azco time on the community channel was her way of fixing this problem. To me, this would seem to be a strong counter for Randy to give to Marion because it provides an even deeper look into why the mayor did in fact do this for the sake of keeping diversity (diversity in programming being the fix to decreased viewership). Could you please clarify why my reasoning is wrong here? Thanks so much in advance for your help!
 Jeremy Press
PowerScore Staff
  • PowerScore Staff
  • Posts: 1000
  • Joined: Jun 12, 2017
|
#76634
Hi Bea,

Good question!

The problem in your line of thought is that you went beyond what the answer choice said to an assumption that can't be safely made. You said that, "if the number of people watching Mega Cable's programming has decreased, then perhaps the mayor thought that that might be ameliorated by ensuring that a diverse array of news programming is available to peak interest in potential viewers." I can't say that's impossible! But do I have any evidence in the argument or the answer choice of that possibility? No. And it's also entirely possible that the mayor doesn't know about the viewership problem answer choice E describes. So where does that leave me with answer choice E? Very uncertain about its effect. In fact, I can't know the effect of answer choice E on Marion's argument without those additional facts. I want the answer choice to weaken on its own (without having to put more information into it).

Keep it simple: if the answer choice doesn't do the job as stated, don't stretch to make it a better answer! Make it do the work on its own :-D .

Hope this helps!

Jeremy
User avatar
 yenisey
  • Posts: 19
  • Joined: Oct 14, 2021
|
#104643
I don't understand here how the answer choice C is incorrect. "Azco’s news coverage of the mayor has never been judged to be biased by an impartial, independent organization." I can see that answer choice B is way stronger than C but I can't see why C can't strengthen the argument .
 Adam Tyson
PowerScore Staff
  • PowerScore Staff
  • Posts: 5153
  • Joined: Apr 14, 2011
|
#104661
First things first here, yenisey, and that is that we are not trying to strengthen an argument. Rather, we are trying to weaken one. The question requires us to counter what Marion said. That means we need some evidence that the Mayor was not necessarily acting based on the crude motive cited by Marion, that the Mayor was just a political supporter.

So what if no independent organization has found any bias? Have they looked? And so what if there was no bias on the channel? It still could be that the Mayor is rewarding her supporter, couldn't it? As we said in our explanation at the top of this thread, this answer just isn't relevant to the question of what motivated the Mayor to do what she did. And if she did the exact same thing in a different situation where the stated motive could not have been present (the effect without the cause) that certainly weakens the claim that the bad motive was the true cause!

Get the most out of your LSAT Prep Plus subscription.

Analyze and track your performance with our Testing and Analytics Package.